Sol'n to PDE Integral: \frac{2b^2a}{3}

  • Thread starter Thread starter Cyrus
  • Start date Start date
  • Tags Tags
    Integral Pde
Click For Summary
The discussion centers on the evaluation of the integral of the function \(\phi_x^2\), where \(\phi_x = b - \frac{b}{a}|x|\) for \(|x| > a\) and zero otherwise. Strauss claims the integral equals \((\frac{b}{a})^2 2a\), but the original poster believes there is an error, asserting that the correct evaluation leads to \(\frac{2b^2a}{3}\). They derive this by integrating a specific expression between the limits of \(0\) and \(a\) and noting the symmetry of the absolute value function. After some confusion regarding the integration limits, the poster confirms they arrive at the same result as Strauss. The discussion highlights potential discrepancies in the PDE book's calculations.
Cyrus
Messages
3,237
Reaction score
17
My PDE book does the following:

\int \phi_x^2 dx

Where,

\phi_x = b-\frac{b}{a} |x|

for |x|> a and x=0 otherwise.

Strauss claims:

\int \phi_x^2 dx = ( \frac{b}{a} ) ^2 2a

However, I think there is a mistake. It can be shown that:

\frac{-3a}{b}(b- \frac{b|x|}{a})^3 is a Soln. Evaluate this between 0<x<a and you get:

\frac{b^2 a}{3}

Because the absolute value function is symmetric, its twice this value:

\frac{2b^2 a}{3}

Unless I goofed, I think the book is in error.

*Note: Intergration is over the whole real line.
 
Last edited:
Physics news on Phys.org
I think you must mean |x|<a, right? Otherwise the integral isn't defined. I tried integrating (b-bx)^2 from 0 to a and I don't get anything close to either answer. Can you clarify?
 
O crap, its b-b/a|x| sorry. See above I fixed it.
 
Then I'm getting the same result as you.
 
Question: A clock's minute hand has length 4 and its hour hand has length 3. What is the distance between the tips at the moment when it is increasing most rapidly?(Putnam Exam Question) Answer: Making assumption that both the hands moves at constant angular velocities, the answer is ## \sqrt{7} .## But don't you think this assumption is somewhat doubtful and wrong?

Similar threads

  • · Replies 5 ·
Replies
5
Views
2K
Replies
3
Views
2K
  • · Replies 4 ·
Replies
4
Views
2K
  • · Replies 105 ·
4
Replies
105
Views
6K
Replies
4
Views
2K
  • · Replies 8 ·
Replies
8
Views
2K
  • · Replies 22 ·
Replies
22
Views
3K
Replies
2
Views
1K
  • · Replies 10 ·
Replies
10
Views
2K
  • · Replies 8 ·
Replies
8
Views
2K